Can someone help with an improper integral problem
Favorites|Homepage
Subscriptions | sitemap
HOME > > Can someone help with an improper integral problem

Can someone help with an improper integral problem

[From: ] [author: ] [Date: 12-11-06] [Hit: ]
= 4.I hope this helps!......
my problem asks to use integration by parts(and L'Hopital's Rule) to show whether the following improper integral diverges or converges. Evaluate the integral if it converges.
Here is the problem and the graph:
http://postimage.org/image/58r2vv7uz/

Could really use someone's help in figuring out how to solve this

-
Let u = x, dv = e^(-x/2) dx
du = 1 dx, v = -2e^(-x/2).

So, ∫(x = 0 to ∞) xe^(-x/2) dx
= lim(t→∞) ∫(x = 0 to t) xe^(-x/2) dx
= lim(t→∞) [-2xe^(-x/2) {for x = 0 to t} - ∫(x = 0 to t) -2e^(-x/2) dx]
= lim(t→∞) [-2xe^(-x/2) - 4e^(-x/2)] {for x = 0 to t}
= lim(t→∞) -2(x + 2)/e^(x/2) {for x = 0 to t}
= [lim(t→∞) -2(t + 2)/e^(t/2)] + 4
= [lim(t→∞) -2/((1/2)e^(t/2))] + 4, by L'Hopital's Rule
= 0 + 4
= 4.

I hope this helps!
1
keywords: with,Can,someone,integral,improper,problem,an,help,Can someone help with an improper integral problem
New
Hot
© 2008-2010 http://www.science-mathematics.com . Program by zplan cms. Theme by wukong .